Đến nội dung

tranductucr1

tranductucr1

Đăng ký: 28-04-2015
Offline Đăng nhập: 16-06-2019 - 22:46
*****

#711381 $2-10-14$

Gửi bởi tranductucr1 trong 22-06-2018 - 08:40

Một bài toán hay và hóc búa :

Một tiến sĩ độc ác Schrodinger đã phát minh tia phóng xạ kích thích tăng trưởng với mục đích tạo thành một binh đoàn mèo khổng lồ tấn công thành phố

attachicon.gifCapture.PNG

Để ngăn chặn ý đồ này, bạn cùng nhóm đặc vụ tiến vào căn phòng thí nghiệm của ông ta nhưng không may, bạn đã rơi vào bẫy, và căn phòng đổ sụp.  Schrodinger đã trốn sang một căn phòng khác và ngừng hệ thống. May sao, nhóm đặc vụ của bạn rất chuyên nghiệp và đặc vụ Delta đã khởi động lại hệ thống, trong khi đó, đặc vụ Alpha tìm kiếm mã mở khóa cho căn phòng tiếp theo và anh đã tìm được : $2-10-14$

attachicon.gifCapture2.PNG

Tất cả những điều mà bạn cần làm để thoát khỏi đó là nhập mã vào hệ thống. Nhưng có một vấn đề : hệ thống bị phá hỏng nên chỉ có 3 nút hoạt động : $+5$; $+7$ và $\sqrt{}$ và bạn phải tạo thành các số "2","10" và "14" theo thứ tự đó. Bạn có thể biểu diễn các số khác nhau nhưng không thể khởi động lại dãy số nên một khi bạn đạt được đến số "2" thì bạn phải tiếp tục biểu diễn "10" theo số "2" và "14" theo số "10". Tuy nhiên còn một vấn đề khác mà đặc vụ Delta giải thích cho bạn : Nếu có 2 số giống nhau hoặc một số $\geq60$ hoặc một số không tự nhiên xuất hiện trên màn hình hiển thị, căn phòng sẽ phát nổ. Hiện tại màn hình đang hiển thị số "0"

( Giả sử màn hình đang hiển thị số "19" ấn $+5$ sẽ được "24", ấn $+7$ được "31", ấn $+5$ được "36" và ấn $\sqrt{}$ được "6" )

attachicon.gifCapture3.PNG )

VẬY BẠN SẼ LÀM THẾ NÀO ĐỂ THOÁT KHỎI CĂN PHÒNG ?

Nguồn : Google  :D  

- Lao Hac and Korkot

p/s : Sorry mọi người em edit hơi tệ :D

 

Do có một số anh, chị chưa hiểu về đề nên em xin giải thích ạ :)

1. Có 2 số giống nhau 

Ví dụ sau khi biến đổi ra số "12" chẳng hạn. Tiếp tục biến đổi một hồi sau mà số "12" lại xuất hiện một lần nữa trên màn hình hiển thị thì phòng phát nổ

2. Có số $\geq60$

Đơn giản là nếu biến đổi ra số $\geq60$ thì phòng phát nổ . Ví dụ ấn 12 lần + 5 chẳng hạn

3. Có số " không tự nhiên"

Có nghĩa là biến đổi thành số thập phân ý ạ :) > Giả sử $\sqrt{12} = 3.464......$

3 TH đó sẽ khiến phòng phát nổ :)

LƯU Ý: CHỈ ĐƯỢC $+5$; $+7$ hoặc $\sqrt{}$

Mọi người khi giải chỉ cần ghi dãy số biến đổi, không cần ghi quy trình tạo nên dãy số đó :)

 

ĐƠN GIẢN HÓA BÀI TOÁN

Từ một số 0, chỉ được $+5, +7, \sqrt{}$ thì làm thế nào để tạo ra một dãy số trong đó có chứa các số "2" "10" và "14". Biết rằng 3 số này phải được sắp xếp theo thứ tự 2 -10 - 14, sao cho dãy số đó không có 2 số trùng lặp, không có số nào $>60$ và tất cả đều là các số tự nhiên

 

VẤN ĐỀ MỞ RỘNG :

Có bao nhiêu cách để tạo ra mật khẩu ?

$0,5,12,19,26,31,36,6,11,16,4,2,9,3,10,17,22,29,34,39,44,49,7,14 $




#690575 Tìm tham số $m$ để cắt đồ thị hàm số $y=\frac{x+m...

Gửi bởi tranductucr1 trong 15-08-2017 - 10:58

Đường thẳng $y=2x+1$ cắt đồ thị hàm số $y=\frac{x+m}{x-1}$

$\Leftrightarrow$ phương trình $(2x+1)(x-1)=x+m$ có nghiệm khác $1$

$\Leftrightarrow$ phương trình $2x^2-2x-m-1=0$ có $2$ nghiệm phân biệt

$\Leftrightarrow \Delta '=1+2m+2=2m+3> 0\Leftrightarrow m> -\frac{3}{2}$

$m=\frac{-3}{2}$ vẫn đúng !!

*** Tại sao phần màu đỏ lại tương đương với nhau ? 




#690573 Tìm phép tịnh tiến biến $y=\dfrac{1}{x-2}$...

Gửi bởi tranductucr1 trong 15-08-2017 - 10:44

Cho $(H)$: $y=\dfrac{1}{x-2}$

Tìm phép tịnh tiến biến $(H)$ thành $y=\dfrac{x-3}{x+7}$.

chỉ dùng phép tịnh tiến thôi thì sao biến được




#677881 Đề thi HSG lớp 9 tỉnh Khánh Hòa 2016-2017

Gửi bởi tranductucr1 trong 18-04-2017 - 17:02

17953026_1886916034924948_14283146249250

Bài 6 . Một ví dụ kinh điển 
gọi $A,B,C,D,E,F$ là các điểm đại diện cho 6 người nói trên, 2 điểm bất kì nối với nhau bằng hai màu xanh hoặc đỏ (màu xanh chứng tỏ hai người đó quen nhau, màu đỏ là ngược lại). Ta đưa bài toán về chứng minh tồn tại tam giác có ba cạnh cùng màu 

  Ta có: xét các cạnh AB,AC,AD,AE,AF theo nguyên tắc Dirichlet: Tồn tại 3 cạnh cùng màu. Giả sử đó là màu xanh, và 3 cạnh đó là AB,AC,AD
Xét tam giác BCD
+Nếu 3 cạnh tam giác là màu đỏ ta có ngay dpcm
+Nếu trong 3 cạnh tam giác tồn tại ít nhất một cạnh màu xanh, giả sử đó là BC. Ta có ngay tam giác ABC thỏa mản điều kiện 
Tóm lại: Đa giác bất kì luôn có 1 tam giác "trùng màu"(Đpcm)




#675979 Tìm số tự nhiên $n$ thỏa mãn:

Gửi bởi tranductucr1 trong 02-04-2017 - 10:17

Tìm số tự nhiên $n$ thỏa mãn: 

  $a^{n}(b-c)+b^{n}(c-a)+c^{n}(a-b)$ chia hết cho $a^{2}+b^{2}+c^{2}+ab+bc+ca$.

         với $a,b,c$ là các số thực bất kì.

nếu đặt $f(a,b,c)=a^{n}(b-c)+...$

ta thấy $f(a,b,c)$ sẽ có dạng  $f(a,b,c)=(a^{4}(b-c)+b^{4}(c-a)+c^{4}(c-a))g(a,b,c)$

đặt $g(a,b,c)=a^k+p(a,b,c)=b^k+q(a,b,c)=c^k+r(a,b,c)$
=>$f(a,b,c)=a^{4k}(b-c)+b^{4k}(c-a)+c^{4k}(a-b)+p(a,b,c)(b-c)+q(a,b,c)(c-a)+r(a,b,c)(a-b)$
để $f(a,b,c)$ có dạng $a^{n}(b-c)+b^{n}(c-a)+c^{n}(a-b)$ với mọi số thực $a,b,c$ thì 
$
p(a,b,c)(b-c)+q(a,b,c)(c-a)+r(a,b,c)(a-b) $ đồng nhất với 0

=> $p(a,b,c) \equiv q(a,b,c) \equiv r(a,b,c) \equiv 0$ 
=> $g(a,b,c)=a^k=b^k=c^k$ => k=0$
=> $g(a,b,c) \equiv 1$ 
$n=4$ là số duy nhất thỏa mãn đề toán 




#675720 Tìm số hạng tổng quát của dãy số un và tính tổng $S = \sum\lim...

Gửi bởi tranductucr1 trong 30-03-2017 - 17:52

Cho dãy số (un): $$\left\{ \begin{array}{l} {u_1} = \sqrt 2 \\ {u_{n + 1}} = \sqrt {4{u_n}^2 + 3} \end{array} \right.$$ với n>=1.

a) Tìm un

b) Tính $$S = \sum\limits_{i = 1}^{2016} {{u_i}^2} $$.

Đặt $y_n=u_n^2$
ta có $y_1=2$ và $y_{n+1}=4y_n+3$ 

=> $y_n=4^{n-1}y_1+3\frac{4^{n-1}-1}{4-1}=3*4^{n-1}-1$ 
=> $u_n=\sqrt{3*4^{n-1}-1}$ 

ta có $S=y_1+y_2+..+y_{2016}$ <=> $S=3(1+4+...+4^{2015})-2016=4^{2016}-2017$
 




#675703 CMR $x^2 +y^4 +z^6 ≤ 2$

Gửi bởi tranductucr1 trong 30-03-2017 - 12:47

Trong $3$ số $x,y,z$ có ít nhất $2$ số cùng dấu. Giả sử $ x,y \geq 0$

$\rightarrow z=-x-y \geq 0$. Vì $-1 \geq x,y,z \geq 1$ $\rightarrow x^2+y^4+z^6 \geq |x| +|y|+|z| \geq 2$

Dấu $"="$ xảy ra khi $x=0;y=1;z=-1$

bài làm  của bạn có nhiều mâu thuẫn chổ phần màu đỏ ? 

cách làm của tôi

ta có $x^2+y^4+z^6 \leq x^2+y^2+z^2=-2(xy+yz+xz) \leq 2$
vì ta luôn có $(1-x)(1-y)(1-z)+(x+1)(y+1)(z+1) \geq 0 \leftrightarrow xy+xz+yz \geq -1$




#671715 Cho $\frac{x}{a}+\frac{y}{b...

Gửi bởi tranductucr1 trong 15-02-2017 - 19:36

mình nghĩ đề là 

Cho $\frac{x}{a}+\frac{y}{b}+\frac{z}{c}=0$ và $\frac{a}{x}+\frac{b}{y}+\frac{c}{z}=2$. Tính giá trị:$A=\frac{a^{2}}{x^{2}}+\frac{b^{2}}{y^{2}}+\frac{c^{2}}{z^{2}}$

Ta có $$A=\frac{a^2}{x^2}+\frac{b^2}{y^2}+\frac{c^2}{z^2} =(\frac{a}{x}+\frac{b}{y}+\frac{c}{z})^2-2\frac{abz+bcx+acy}{xyz}=4-2\frac{abz+bcx+acy}{xyz}$$
Ta lại có $$\frac{x}{a}+\frac{y}{b}+\frac{z}{c}=0$$ 
=> $$abz+bcx+acy=0$$

Vậy $$A=4$$




#671524 tìm n $\in N$ biết $1+2^{2}+3^{2}+.....

Gửi bởi tranductucr1 trong 13-02-2017 - 20:49

bạn có thể nói rõ hơn tại sao lại có công thức đó không?

 

C1 sử dụng quy nạp cái này đơn giản 
C2 ta có 
$S=1^2+2^2+...+n^2=1*(2-1)+2(3-2)+..+n(n+1-1)=1*2+2*3+3*4+...+n(n+1)-(1+2+3+4+...+n)$(3) 

ta có $1+2+3+...+n=\frac{n(n+1)}{2}$(2)
và $1*2+2*3+...+n(n+1)=\frac{1*2*3+2*3*3+...+n(n+1)*3}{3}=\frac{1*2*3+2*3*(4-1)+3*4*(5-2)+...+n(n+1)((n+2)-(n-1))}{3}=\frac{1*2*3-2*3*4+...+n(n+1)(n+2)-(n-1)n(n+1)}{3}=\frac{n(n+1)(n+2)}{3} $ (1)

thay (1) (2) vào (3) ta được 

$S=\frac{n(n+1)(n+2)}{3}-\frac{n(n+1)}{2}=\frac{n(n+1)(2n+1)}{6}$

Vừa nhớ ra một cách hay dùng để chứng minh tổng quát cho loại bài toán này :V 
 gọi $S_k=1^k+2^k+...+n^k$
ta có
$2^3=(1+1)^3=1^3+3*1^2+3*1+1^3$
$3^3=(1+1)^3=1^3+3*2^2+3*2+2^3$
...
$(n+1)^3=1^3+3*n^2+3*n+n^3$
=> $2^3+3^3+...+n^3+(n+1)^3=(1^3+1^3+...+1^3)+3(1^2+2^2+...+n^2)+3(1+2+..+n)+1^3+2^3+...+n^3$
=> $(n+1)^3=n+3S_2+3S_1+1$
=>$S_2=\frac{(n+1)^3-1-3S_1-n}{3}$
ta có kết quả quen thuộc $S_1=\frac{n(n+1)}{2}$
=>$S_2=\frac{n(n+1)(2n+1)}{6}$




#671517 tìm n $\in N$ biết $1+2^{2}+3^{2}+.....

Gửi bởi tranductucr1 trong 13-02-2017 - 20:35

bạn có thể nói rõ hơn tại sao lại có công thức đó không?

C1 sử dụng quy nạp cái này đơn giản 
C2 ta có 
$S=1^2+2^2+...+n^2=1*(2-1)+2(3-2)+..+n(n+1-1)=1*2+2*3+3*4+...+n(n+1)-(1+2+3+4+...+n)$(3) 

ta có $1+2+3+...+n=\frac{n(n+1)}{2}$(2)
và $1*2+2*3+...+n(n+1)=\frac{1*2*3+2*3*3+...+n(n+1)*3}{3}=\frac{1*2*3+2*3*(4-1)+3*4*(5-2)+...+n(n+1)((n+2)-(n-1))}{3}=\frac{1*2*3-2*3*4+...+n(n+1)(n+2)-(n-1)n(n+1)}{3}=\frac{n(n+1)(n+2)}{3} $ (1)

thay (1) (2) vào (3) ta được 

$S=\frac{n(n+1)(n+2)}{3}-\frac{n(n+1)}{2}=\frac{n(n+1)(2n+1)}{6}$




#664058 $\boxed{Topic}$ÔN THI HỌC SINH GIỎI TOÁN 9 - NĂM HỌC...

Gửi bởi tranductucr1 trong 07-12-2016 - 12:58

Một bài bất đẳng thức, một bài phương trình. Mong mọi người tham gia đóng góp bài tập và giải bài

$\boxed{1}$ Ba số a, b, c thỏa mãn $0\leq a, b, c\leq 2$ và a + b + c = 3. Tìm giá trị lớn nhất của tổng A = a2 + b2 + c2.

$\boxed{2}$ Giải phương trình $\sqrt[3]{25x(2x^2+9)}=4x+\frac{3}{x}$

$(2-a)(2-b)(2-c)+abc \geq 0$ <=> $ab+bc+ac \geq 2 $

$A=(a+b+c)^2-2(ab+bc+ac) \leq 9-2*2=5$




#662341 Bất đẳng thức qua các kì thi toán quốc tế

Gửi bởi tranductucr1 trong 18-11-2016 - 19:36

 

$\S$ Bất đẳng thức qua các kì thi toán quốc tế

 

$\bigstar$ Lời nói đầu: 
Đây là topic về các bài bất đẳng thức qua các kì thi IMO, Olympiad, .... , các bài toán nổi tiếng và các bài toán xuất hiện trên các tạp chí toán học nổi tiếng.

 

$\bigstar$ Ghi chú: 
Các bạn đọc kĩ ghi chú trước khi đăng bài giải và đăng bài giúp topic phát triển mạnh mẽ
$\bullet$ Bài đăng đúng chủ đề, không spam,....
$\bullet$ Bài đăng phải đánh số thứ tự bài 1, 2, .... khi đăng cần chú ý .
$\bullet$ Đáp án đăng phải rõ ràng đúng, bài đăng sai nếu phát hiện phải chỉ rõ ra chỗ sai không được nói bừa
$\bullet$ Không sử dụng từ ngữ thô tục, không chửi nhau, không nói bậy.

Cùng chung tay xây dựng topic phát triển mạnh mẽ !!!  :ukliam2:  :ukliam2:  :ukliam2: 
 


  • Bài 1 : [IMO 1964]

Cho a, b, c là độ dài 3 cạnh của 1 tam giác. Chứng minh rằng:

$a^{2}(b+c-a)+b^{2}(c+a-b)+c^{2}(a+b-c)\leq 3abc$

  • Bài 2 : [IMO 1983]

Cho a, b, c là độ dài ba cạnh của 1 tam giác. Chứng minh rằng:

$a^{2}b(a-b)+b^{2}c(b-c)+c^{2}a(c-a)\geq 0$

  • Bài 3 : [UK 1983] 

Chứng minh rằng $\forall$ a, b, c , d > 0. Ta có:

$\frac{ab}{a+b}+\frac{cd}{c+d}+\frac{ef}{e+f}\leq \frac{(a+c+e)(b+d+f)}{a+b+c+d+e+f}$

  • Bài 4 : [China 1984] 

Cho n $\geq$ 2 và các số thực dương $a_{1},a_{2},...,a_{n}$. Chứng minh rằng: 

$\frac{a_{1}^{2}}{a_{2}}+\frac{a_{2}^{2}}{a_{3}}+...+\frac{a_{n-1}^{2}}{a_{n}}+\frac{a_{n}^{2}}{a_{1}}\geq a_{1}+a_{2}+...+a_{n}$

  • Bài 5 : [IMO 1984]

Cho x, y, z $\geq$ 0 và x + y + z = 1. Chứng minh rằng: 

$0\leq xy+yz+zx-2xyz\leq \frac{7}{27}$ 

  • Bài 6 : [Russia 1984]

Chứng minh rằng với mọi a, b > 0 ta luôn có:

$\frac{(a+b)^{2}}{2}+\frac{a+b}{4}\geq a\sqrt{b}+b\sqrt{a}$

  • Bài 7 : [UK 1986]

Cho x + y + z = 0 và $x^{2}+y^{2}+z^{2}=6$. Tìm Giá trị lớn nhất của: 

$x^{2}y+y^{2}z+z^{2}x.$

  • Bài 8 : [IMO Shortlist 1987]

Cho x, y, z $\in \mathbb{R}$ và $x^{2}+y^{2}+z^{2}=2$. Chứng minh rằng: 

$x+y+z\leq xyz+2$

  • Bài 9 : [Kvant 1988]

$3+a+b+c+\frac{1}{a}+\frac{1}{b}+\frac{1}{c}+\frac{a}{b}+\frac{b}{c}+\frac{c}{a}\geq 3.\frac{(a+1)(b+1)(c+1)}{1+abc}$

Cho a, b, c > 0. Chứng minh rằng:

 

  • Bài 10 : [Russia 1989]

Cho x, y, z $\in \mathbb{R}$ và xyz(x + y + z) = 1. Tìm giá trị nhỏ nhất của:

$\left ( x+y \right )\left ( y+z \right )$

  • Bài 11 : [IMO Shortlist 1990] 

Cho a, b, c, d > 0 và ab + bc + cd +da = 1. Chứng minh rằng:

$\sum \frac{a^{3}}{b+c+d}\geq \frac{1}{3}$

  • Bài 12 : [Russia 1990]

Chứng minh rằng với mọi x ta có: 

$x^{4}>x-\frac{1}{2}$

  • Bài 13 : [UK 1990]

Chứng minh rằng với mọi x, y, z ta luôn có bất đẳng thức sau: 

$\sqrt{x^{2}-xy+y^{2}}+\sqrt{y^{2}-yz+z^{2}}\geq \sqrt{z^{2}+zx+x^{2}}$

  • Bài 14 : [APMO 1991]

Cho $a_{i}>0,b_{i}>0$ $\forall i=\overline{1,n}$ và $a_{1}+a_{2}+...+a_{n}=b_{1}+b_{2}+...+b_{n}$

  • Bài 15 : [Mongolia 1991]

Cho a, b, c lá các số thực thoả mãn $a^{2}+b^{2}+c^{2}=2$. Chứng minh rằng:

$\prod (1+a)\geq 8\prod (1-a)$

  • Bài 16 : [Russia 1991] 

Cho x, y, z là các số thực không âm. Chứng minh bất đẳng thức sau: 

$\frac{(x+y+z)^{2}}{3}\geq x\sqrt{yz}+y\sqrt{xz}+z\sqrt{xy}$

  • Bài 17 : [Viet Nam 1991]

Chứng minh rằng $\forall x\geq y\geq z>0$ ta có :

$\sum \frac{x^{2}y}{z}\geq x^{2}+y^{2}+z^{2}$

  • Bài 18 : [Poland 1992]

Chứng minh rằng với mọi số thực a, b, c ta luôn có: 

$\prod (b+c-a)^{2}\geq \prod (b^{2}+c^{2}-a^{2})$

  • Bài 19 : [Russia 1992]

Cho x, y, z > 0. Chứng minh rằng: 

$x^{4}+y^{4}+z^{4}\geq 2\sqrt{2}xyz$

  • Bài 20 : [United Kingdom 1992]

Cho x, y, z, w là các số thực dương. Chứng minh rằng : 

$\frac{12}{x+y+z+w}\leq \sum_{sym}^{} \frac{1}{x+y}\leq \frac{3}{4}\left ( \frac{1}{x}+\frac{1}{y}+\frac{1}{z}+\frac{1}{w} \right )$

 

Khi nào rảnh mình sẽ đăng tiếp. 

 

Russia 92
$x^4+y^4+z^4 \geq x^4+2(yz)^2 \geq 2\sqrt{2}xyz $

Russia 89

$(x+y)(y+z)=y^2+xy+yz+xz=y(x+y+z)+xz \geq 2\sqrt{xyz(x+y+z)}=2$




#662339 $a(cosx-1)+b^{2}+1-cos(ax+b^{2})=0$

Gửi bởi tranductucr1 trong 18-11-2016 - 19:21

Tìm a, b sao cho $a(cosx-1)+b^{2}+1-cos(ax+b^{2})=0$  với mọi x.

Cho $x=0$ ta có $cos(b^2)=b^2+1 \geq 1$ mà $cos (b^2) \leq 1$ => $b=0$

Thay $b=0$ ta có $1-a=cos(ax)-cosx$ xét hàm $f(x)=cos(ax)-cosx=1-a$ là hằng số với mọi x thuộc $R$ 
Lập luận theo định lý Lagrange ta được $f'(x)=a(sinx-sin(ax))=0$ có vô số nghiệm 
vậy $sinx=sin(ax)$ có vô số nghiệm từ đó ta  có $a=1$




#660157 Tìm số có 4 chữ số abcd thỏa mãn

Gửi bởi tranductucr1 trong 01-11-2016 - 12:54

Tìm số có 4 chữ số abcd thỏa mãn :

$\left\{\begin{matrix} a+b=c.d & & \\ c+d=a.b& & \end{matrix}\right.$

không mất tính tổng quát giả sử $c \geq d $#=== một nhận xét nhỏ là $a,b,c,d$ không thể bằng không
ta có $d^2 \leq cd =a+b \leq 18 $ 
=> $d \leq 4$  
Xét $d=1$ 
$a+b=c$ và $c+1=ab$ từ đó ta có $(a-1)(b-1)=2$ từ đó ta có $a=2;b=3$ hoặc $a=3;b=2$
xét $d=2$
$a+b=2c$ và $c+2=ab$ từ đó ta có $a+b=2ab-4$ => $(2a-1)(2b-1)=9$=> $a=2;b=2$,$a=1;b=5$ $a=5;b=1$ 
Xét $d=3$

$a+b=3c$ và $c+3=ab$ từ đó ta có $a+b=3ab-9$ =>$(3a-1)(3b-1)=28$ => $a=5;b=1$ va các hoán vị 
Xét $d=4$ => $a+b=4ab-16$ từ đó ta có $(4a-1)(4b-1)=63$ vô nghiệm vì $VT \neq VP (mod 4)$ 

Tog lại ta có các số $2351,2315,3251,3215,2222,5132,5123,1523,1532$
dài !!! 




#659236 $P=\sum z^2 \left ( \dfrac{1}{x-1}+...

Gửi bởi tranductucr1 trong 25-10-2016 - 05:44

Cho $x,y,z>1$. Tìm Min
$$P=\sum z^2 \left ( \dfrac{1}{x-1}+\dfrac{1}{y-1} \right )$$

áp dụng C-S ta được : 
$P \geq \sum \frac{4z^2}{x+y-2} \geq 4\frac{(x+y+z)^2}{2(x+y+z)-6}=\frac{2t^2}{t-3}$ (với $t=x+y+z$)
Khảo xác hàm $f(t)=\frac{2t^2}{t-3}$ với $t >3$ ta được 

$f(t) \geq 24$ 
vậy $min P=24$ tại $x=y=z=2$